28 Rosh Review 2018

Lakukan tugas rumah & ujian kamu dengan baik sekarang menggunakan Quizwiz!

Question: What ethnic group in the US has the highest incidence of Kawasaki disease?

Answer: Americans of Asian and Pacific island descent. Rapid Review Kawasaki Disease Patient will be a child < 4 years old With a history of high fever for 5 days Complaining of conjunctivitis, rash, adenopathy, strawberry tongue, hand/feet edema, fever Treatment is IVIG + aspirin Comments: #1 cause of pediatric acquired heart disease, risk for coronary artery aneurysm Mnemonic: CRASH and burn: Conjunctivitis, Rash, Adenopathy, Strawberry tongue, Hand/feet edema, Fever

Question: What is the name of the reaction that manifests as fatty liver and cerebral edema associated with acute varicella infection and salicylates?

Answer: Reye or Reye's syndrome is a potentially fatal syndrome associated with the ingestion of aspirin by children with chickenpox.

Question: How many café-au-lait macules are required for a diagnosis of neurofibromatosis?

Answer: Six.

Question: What is the most common cause of acromegaly?

Answer: Somatotroph adenoma of the pituitary. Rapid Review Acromegaly Patient will be complaining of increased head, glove, or shoe size PE will show coarse facial features, oily skin, visual field deficits, diabetes Labs will show increased IGF-1 Most commonly caused by pituitary adenoma Treatment is transphenoid resection

Question: Hypertrophic cardiomyopathy in a young athlete can commonly lead to which disastrous result?

Answer: Sudden cardiac arrest and death. Rapid Review Hypertrophic Cardiomyopathy Asymmetric LV septal wall hypertrophy → outflow obstruction Autosomal dominant (familial form) Young patient Exertional syncope Sudden cardiac death S4 gallop Midsystolic murmur (↑ as preload ↓) Rx: ßBs or CCBs

Question: What medications constitute triple therapy for Helicobacter pylori?

Answer: Clarithromycin, Amoxicillin (or metronidazole) and a proton pump inhibitor. Rapid Review Gastritis Acute: NSAIDs > alcohol Type A chronic: pernicious anemia Type B chronic : H. pylori

Question: What X-ray finding is seen in patients with sigmoid volvulus?

Answer: Coffee bean sign.

Question: In which trimester is mumps most devastating?

Answer: Congenital infection is rare, but there is increased likelihood of fetal loss if it occurs in the 1st trimester. Rapid Review Mumps Viral paramyxovirus Winter/spring Parotitis Orchitis Deafness Aseptic meningitis

Question: When evaluating for ischemic colitis, in what condition should a colonoscopy not be performed?

Answer: Acute peritonitis.

Question: What are the two most common types of cervical cancer?

Answer: Adenocarcinoma and squamous cell carcinoma.

Question: What are the three most common causes of SBO in the US?

Answer: Adhesions from prior surgeries, hernias and tumors. Rapid Review Small Bowel Obstruction Causes: adhesions (pelvic surgery) > tumor > hernia (inguinal) Proximal: bilious vomiting Distal: feculent vomiting High pitched bowel sounds X-ray: dilated bowel, air fluid levels, "stack of coins" or "string of pearls" sign Modality of choice: CT NGT, surgery

Question: What are the most common risk factors for an intracerebral hemorrhage?

Answer: Age over 55, hypertension, and alcohol consumption.

Question: When prescribing metronidazole, patients should be counseled to avoid consumption of what substance?

Answer: Alcohol should be avoided due to the disulfiram-like reaction that occurs when taken with metronidazole. Rapid Review Trichomoniasis Patient will be a woman complaining of malodorous vaginal discharge PE will show frothy, green/yellow discharge, "Strawberry cervix" Labs will show pH >5, flagellated, motile, pear shaped Diagnosis is made by wet mount Most commonly caused by Trichomonas vaginalis Treatment is metronidazole

What is aspiration pneumonia?

An alveolar space infection resulting from the inhalation of pathogenic material from the oropharynx. Aspiration pneumonitis is chemical injury, usually from inhalation of gastric contents. Can lead to pneumonia, which is when antibiotics would be indicated. Usually unilateral.

Question: How long does it take for a subconjunctival hemorrhage to resolve?

Answer: 10-14 days. Rapid Review Subconjunctival Hemorrhage Coughing, sneezing, minor trauma Under conjunctiva and stops at limbus Reassurance

Question: What is the usual timeframe between exposure to HIV and development of clinical symptoms?

Answer: 2-4 weeks. Rapid Review HIV Infection Patient will be complaining of abrupt onset of a viral-like illness, fever, malaise, sore throat, headache, arthralgias, anorexia, nausea, vomiting, and rash Diagnosis is made by nucleic acid amplification tests Treatment is HAART

Question: What is the rule of nines?

Answer: A quick reference to approximate the total body surface area that has been burned.

Question: What are 2 concerning complications of GERD?

Answer: Barrett's esophagus and esophageal adenocarcinoma. Rapid Review Gastroesophageal Reflux Disease (GERD) Heartburn Nocturnal cough/asthma Acid injury to enamel Chronic GERD → Barrett esophagus → ↑ neoplasm risk Empiric trial of PPIs

Question: How many days after implantation is beta-hCG detectable?

Answer: Beta-hCG is secreted from the time of implantation and is detectable about 7-8 days after fertilization. Rapid Review Ectopic Pregnancy Patient will be a woman With a history of prior ectopic, PID, tubal surgery, IUD Complaining of vaginal bleeding, abdominal pain, amenorrhea PE will show adnexal tenderness or unexplained hypotension Labs will show positive pregnancy test and lower than expected serum ß-hCG levels Diagnosis is made by ultrasound Most commonly located in a fallopian tube Treatment is MTX or surgery

Question: Name some common psychiatric comorbidities in perpetrators of domestic violence?

Answer: Borderline and antisocial personality disorders, bipolar disorder, schizophrenia and substance abuse. Rapid Review Intimate Partner Violence Head, face, neck injuries Stated history does not correlate with exam Directly ask patient Victims at highest risk during attempts to terminate relationship Management: assess immediate risk, document thoroughly, social work referral

Question: Which malignancy is associated with Epstein Barr Virus?

Answer: Burkitt's lymphoma. Rapid Review Infectious Mononucleosis Patient will be complaining of low-grade fever, headache, malaise, severe fatigue PE will show mildly tender lymphadenopathy involving the posterior cervical chain, hepatosplenomegaly Diagnosis is made by heterophile antibody test (monospot test), generalized maculopapular rash following administration of amoxicillin Most commonly caused by Epstein-Barr virus Treatment is self-limiting, refrain from contact sports for four weeks post-infection

Question: Which beta-blockers have a proven mortality benefit in the treatment of heart failure?

Answer: Carvedilol, metoprolol succinate and bisoprolol. Rapid Review Chronic Heart Failure Treatment Lifestyle modifications Diuretics: used for acute pulmonary edema, no mortality benefit ACEIs: ↓ mortality in all classes BBs: ↓ mortality in classes II, III, IV Hydralazine with nitrates: ↓ mortality in African-Americans Digoxin: used in refractory systolic dysfunction, no mortality benefit Spironolactone: ↓ mortality in class III/IV Isolated diastolic dysfunction: HR and BP control Advanced treatments: mechanical assist device, ICD, heart transplantation

Question: What portion of the esophagus is typically involved in primary achalasia?

Answer: Distal two-thirds of the esophagus. Rapid Review Achalasia Patient will be complaining of dysphagia to solids and liquids PE will show absent peristalsis in lower esophagus Barium swallow shows "bird beak" appearance Diagnosis is made by esophageal manometry increased LES pressure

Question: What is the treatment of choice for Rocky Mountain Spotted Fever?

Answer: Doxycycline. Rapid Review Hand, Foot, Mouth Disease (HFMD) Patient will be a child younger than five years of age Complaining of decreased appetite and fever PE will show an oral exanthem plus a macular, maculopapular, or vesicular rash on the hands and feet Most commonly caused by Coxsackie virus A Treatment is supportive

Question: What are the classic "Red Flag" symptoms of esophageal cancer?

Answer: Dysphagia, vomiting, anorexia, weight loss, and symptoms of gastrointestinal-related blood loss. Rapid Review Esophageal Neoplasm Patient will be a man With a history of weight loss or chronic GERD Complaining of progressive dysphagia to solid foods Diagnosis is made by endoscopy with biopsy Most common type adenocarcinoma Comments: usually a complication of GERD/Barrett's esophagus

Question: What is the most common cause of anaphylaxis?

Answer: Foods in children, medications and insect stings in adults.

Question: What medication may be used to facilitate passage of a lower esophageal foreign body prior to endoscopy?

Answer: Glucagon. Rapid Review Schatzki Ring Most common esophageal structural abnormality Fibrous esophageal ring → esophageal stenosis Dysphagia

Question: Which electrolyte abnormalities can cause a long QT syndrome?

Answer: Hypokalemia and hypomagnesemia.

Question: What are the classic hormone level alterations in menopause?

Answer: Increased follicle-stimulating hormone, decreased estrogens and decreased progesterone. Rapid Review Atrophic Vaginitis Patient will be a postmenopausal woman Complaining of dyspareunia, dryness, bleeding, itching PE will show pale, dry, shiny epithelium Most commonly caused by decrease in estrogen Treatment is lubricants, moisturizers, topical estrogen (2nd line)

Question: How does stridor develop?

Answer: It is caused by partial airway obstruction and the resultant turbulent airflow through the upper airway. Rapid Review Tracheitis S. aureus 3-5 years old URI prodrome → suddenly worsens with airway obstruction, toxic appearance CXR: subglottic narrowing Gold standard dx: bronchoscopy ABX, intubation

Question: What is a reasonable hemoglobin A1C in most patients with diabetes mellitus?

Answer: Less than or equal to 7.0%.

Question: What is the difference between albuterol and levalbuterol?

Answer: Levalbuterol contains only the R-enantiomer of albuterol, the active isomer. Rapid Review Bronchiolitis Patient will be an infant Complaining of difficulty breathing PE will show respiratory distress, polyphonic wheezing, and rales CXR will show diffuse infiltrates Diagnosis is made by history and physical exam Most commonly caused by respiratory syncytial virus (RSV) Treatment is supportive care

Question: What is the name of the syndrome that causes hereditary nonpolyposis colorectal cancer, in addition to increasing patients' risk of endometrial cancer, ovarian cancer, stomach cancer, or kidney cancer?

Answer: Lynch syndrome should be suspected in patients with a strong family history of colon or endometrial cancer and may be grounds for more vigilant screening procedures. Rapid Review Endometrial Cancer Patient will be a postmenopausal woman Complaining of abnormal vaginal bleeding Diagnosis is made by transvaginal ultrasound or endometrial biopsy Most common type is adenocarcinoma Treatment is total abdominal hysterectomy and bilateral salpingo-oophorectomy (TAH-BSO)

Question: Which medications commonly used to treat condyloma accuminata should be avoided during pregnancy?

Answer: Podophyllin, podophyllotoxin, and imiquimod. Rapid Review Condyloma Acuminata Patient will be complaining of genital lesions PE will show cauliflower-like lesion Most commonly caused by HPV 6 & 11 Comments: most common STD

Question: What are the most common bacterial pathogens that cause otitis externa?

Answer: Pseudomonas aeruginosa and Staphylococcus aureus. Rapid Review Otitis Externa Patient with a history of swimming or moisture exposure Complaining of malodorous discharge and pruritus PE will show pain with palpation of tragus/pinna Most commonly caused by Pseudomonas aeruginosa Treatment is topical antimicrobials with or without steroids Comments: Necrotizing otitis externa - a complication seen in diabetics/immunocompromised

Question: What bacteria is commonly the cause of otitis externa?

Answer: Pseudomonas aeruginosa. Rapid Review Otitis Externa Patient with a history of swimming or moisture exposure Complaining of malodorous discharge and pruritus PE will show pain with palpation of tragus/pinna Most commonly caused by Pseudomonas aeruginosa Treatment is topical antimicrobials with or without steroids Comments: Necrotizing otitis externa - a complication seen in diabetics/immunocompromised

Question: What are the complications of cryptorchidism?

Answer: Testicular carcinoma, subfertility and testicular torsion. Rapid Review Cryptorchidism Incomplete/improper descent of testis into scrotal sac Most common genitourinary disorder in male children Most spontaneously descend by 4 mos Rx: orchidopexy if between 6 and 12 mos Complications: seminoma, infertility, torsion

Question: What is the classic pathognomonic finding seen on ultrasonography in patients with intussusception?

Answer: The "target" or "bull's eye" sign. Rapid Review Intussusception (Telescoping Bowel) Patient will be a child 5 months - 3 years old Complaining of colicky abdominal pain, vomiting and bloody stools (currant jelly) Diagnosis is made by ultrasound ("target sign") Most commonly caused by a tumor or meckel's diverticulum Treatment is air/contrast enema

Question: The return of what reflex demonstrates the end of spinal shock?

Answer: The bulbocavernosus reflex. This is performed by placing one finger in the rectum and squeezing the glans of the penis or clitoris leading to rectal sphincter contraction. Rapid Review Brown-Sequard Syndrome Patient with a history of penetrating trauma PE will show ipsilateral loss of motor, position and vibration and contralateral loss of pain and temperature Most commonly caused by spinal cord hemisection

Question: What animal most commonly transmits rabies worldwide?

Answer: The dog. Rapid Review Rabies Patient with a history of exposure to raccoons, bats, or skunks Complaining of hydrophobia, agitation, spasms Treatment is wound care (scrubbing), Ig at wound site, vaccination

Question: What is obstipation?

Answer: The inability to pass either stool or flatus for more than 8 hours despite a perceived need. It is highly suggestive of intestinal obstruction. Rapid Review Appendicitis Patient will be complaining of fever, pain that began periumbilical then moved to RLQ, nausea and anorexia PE will show Psoas sign (RLQ pain on extension of right hip), Obturator sign (RLQ pain on internal rotation of flexed right hip), Rovsing sign (right lower quadrant pain when the left lower quadrant is palpated) Diagnosis is made by ultrasound, CT Most commonly caused by fecalith Treatment is surgery

Question: What is the Phalen maneuver?

Answer: The opposing dorsal surfaces of the hands are pressed together with the wrists flexed for 60 seconds. It is positive for median nerve problems if this reproduces or worsens symptoms. Rapid Review Carpal Tunnel Syndrome Durkan's test: reproduction of symptoms with carpal tunnel compression Phalen's sign: reproduction of symptoms with wrist hyperflexion Tinel's sign: reproduction of symptoms with percussion over carpal tunnel Flick sign: patient reports shaking of the hand provides relief NSAIDs, volar splint in neutral position

Question: What are the typical laboratory abnormalities found in patients with myofascial pain syndrome?

Answer: There are no specific lab tests to confirm or refute a diagnosis of myofascial pain syndrome. Rapid Review Myofascial Pain Trigger points Rx: physical therapy, cognitive-behavior therapy, muscle relaxants

Question: What is the most commonly used first line agent to treat hypertension?

Answer: Thiazide diuretics, such as hydrochlorothiazide or chlorthalidone. Rapid Review Asymptomatic Hypertension MC causes of BP elevation: pain, anxiety No workup indicated Outpatient follow-up

Question: How long should a deer tick be attached before prophylactic treatment to prevent Lyme disease is recommended?

Answer: Typically, longer than 36 hours. Rapid Review Lyme Disease NE USA Campers/hikers Ixodes tick harbors Borrelia burgdorferi Stage I: erythema migrans (pathognomonic), viral-like syndrome Stage II: arthritis, myocarditis, bilateral Bell's palsy Stage III: chronic arthritis, chronic encephalopathy Rx: doxycycline CNS/cardiac involvement: IV ceftriaxone Pregnant/children Rx: amoxicillin

Question: What are the most common risk factors for bacterial vaginosis?

Answer: Vaginal pH > 4.5, frequent douching, pregnancy and intrauterine device presence. Rapid Review Bacterial Vaginosis Patient will be complaining of malodorous vaginal discharge PE will show thin, gray/white discharge Labs will show pH > 4.5, clue cells Diagnosis is made by KOH to smear → fishy odor, "whiff test", Amsel Criteria Most commonly caused by Gardnerella vaginalis Treatment is metronidazole

What is the most common personality disorder?

Borderline personality disorder. It is a cluster B personality disorder, usually in females. Increased suicide risk.

A 3-month-old presents with three days of cough, congestion, and increasing work of breathing. On exam, respiratory rate is 60 breaths per minute with moderate subcostal retractions. Air movement is good, but there are scattered rhonchi, crackles, and expiratory wheezes. Heart rate is 145 beats per minute with a regular rate and rhythm and no murmurs. Oxygen saturation is 89% on room air. Which of the following is the next step in management? Initiate supplemental oxygen by nasal cannula Obtain a chest radiograph Trial of nebulized albuterol Trial of nebulized epinephrine

Correct Answer ( A ) Explanation: The above patient has bronchiolitis. His history is classic with initial upper respiratory symptoms of congestion and cough progressing to lower respiratory symptoms such as chest congestion and increased work of breathing. His examination reveals scattered crackles and wheezing, which are also typical findings of bronchiolitis. Bronchiolitis is a virally-induced inflammation of bronchioles that typically occurs in children < 2 years of age. Respiratory syncytial virus (RSV) is the most common cause of bronchiolitis, although others, including rhinovirus, adenovirus, human metapneumovirus, parainfluenza, influenza, and many others can cause bronchiolitis. Bronchiolitics display a wide range of disease severity, and those without hypoxemia, respiratory distress, or dehydration can be managed in the outpatient setting. However, the above patient has an oxygen saturation is below 90%. As per the 2015 American Academy of Pediatrics (AAP) Bronchiolitis Clinical Practice guidelines, qualifies him for initiation of supplemental oxygen by nasal cannula. Additionally, he has increased work of breathing as indicated by subcostal retractions. As such, he may benefit from high flow nasal canula to increase positive end expiratory pressure. As per the 2014 AAP Bronchiolitis Clinical Practice Guideline, neither a trial of nebulized albuterol (C) or trial of nebulized epinephrine (D) is indicated in bronchiolitic patients even if wheezes are present on auscultation. Notably, this recommendation has changed from previous AAP guidelines. Both recommendations against albuterol and nebulized epinephrine are strong recommendations with evidence quality B. Lastly, physicians should not routinely obtain a chest radiograph (B) in infants or children with bronchiolitis. Bronchiolitis is a clinical diagnosis based on history and physical examination and does not require radiographic or laboratory assessment. If radiographs are obtained, non-specific findings include hyperinflation, peribronchiolar cuffing, and atelectasis with volume loss.

A 62-year-old woman presents for her annual well woman exam. She wants to discuss her risk of endometrial cancer, given a personal history of polycystic ovarian syndrome and prolonged tamoxifen use during her 50's. Given these risk factors she should be cautioned that early endometrial cancer normally presents in which of the following manners? Abnormal vaginal bleeding Asymptomatic with an abnormal Pap smear Bloating and frequent urination Marked lower abdominal pain

Correct Answer ( A ) Explanation: Abnormal vaginal bleeding is the presenting symptom in 80% of women with endometrial carcinoma. The most common cancer of the female genital tract, endometrial cancer usually presents in women 50-70 years of age. In addition to a history of tamoxifen use and polycystic ovarian syndrome (which contributes to anovulation), risk factors include obesity, nulliparity, using unopposed estrogen, and a family history of colon cancer. Women with risk factors for endometrial hyperplasia and consequently endometrial cancer can reduce their risk by using cyclic progestin therapy or contraceptives. Any woman with suspicious abnormal menstrual bleeding or post-menopausal vaginal bleeding should undergo endocervical and endometrial sampling. From there, further imaging with transvaginal ultrasonography, cystoscopy, flexible sigmoidoscopy, and chest radiography may be needed to assess for metastasis. The mainstay of endometrial cancer treatment is a total hysterectomy and bilateral salpingo-oophorectomy. Irradiation is used if lymph node involvement is noted or if invasion has extended deep into the myometrium. In early disease, the depth of myometrial involvement is the strongest predictor of survival, with a 98% 5-year survival rate in patients with less than 66% depth of invasion. Presenting as asymptomatic with an abnormal Pap smear (B) is not likely in endometrial cancer. Rather, though atypical endometrial cells are occasionally seen on a Pap smear, most women will have a negative smear. Bloating and frequent urination (C) is characteristic of ovarian cancer, not early endometrial cancer. Marked lower abdominal pain (D) would not be the first symptom of endometrial cancer. Usually women with endometrial cancer do not develop pain until the cancer has metastasized or a superimposed infection has resulted.

A 23-year-old man presents to your clinic with complaints of fever, sore throat, headache and a rash on his chest. While taking his history, the patient denies any new sexual partners but appears uncomfortable answering the question. Physical exam reveals a temperature of 102°F, non-tender cervical and axillary lymphadenopathy, pharyngeal edema without exudate, and a maculopapular rash on the chest and neck. Which of the following is the most likely diagnosis? HIV Mononucleosis Pityriasis rosea Systemic lupus erythematosus

Correct Answer ( A ) Explanation: Acute HIV infection can present with a number of non-specific symptoms that resemble other illnesses. Medical providers need a high degree of clinical suspicion to make the diagnosis. Patients may be reluctant to answer questions honestly about their sexual behavior or drug use or may not perceive their behavior as being high risk and therefore omit important information in the history. This can impede making a correct diagnosis. Patients presenting with fever, rash, pharyngitis, lymphadenopathy and myalgias should have acute HIV infection included in the differential diagnosis. Mononucleosis (B) caused by Epstein-Barr virus presents with similar clinical features to acute HIV infection. Abrupt onset of symptoms, presence of rash and diarrhea, and lack of tonsillar exudates are clinical features seen in acute HIV infection but not in mononucleosis. The rash of pityriasis rosea (C) is similar to that seen in acute HIV infection, however constitutional symptoms are absent. Systemic lupus erythematosus (D) can closely mimic acute HIV infection, however testing will show the presence of antinuclear antibodies which are not found in HIV infection.

A 3-year-old male presents with fever, cough, and difficulty breathing x 1 day. The mother reports that the patient has had 4 days of mild cough, sore throat, and rhinorrhea, and today the patient suddenly appeared much sicker. Immunizations are up to date. Vitals signs are HR 145, RR 30, T 39.6°C, and oxygen saturation 96% on room air. On exam, the patient is ill appearing. The lungs are clear to auscultation, but you note inspiratory and expiratory stridor. After administering nebulized saline, racemic epinephrine, and steroids, the patient's condition fails to improve. A soft tissue radiograph shows minimal subglottic narrowing. Which of the following is the most likely diagnosis? Bacterial tracheitis Croup Epiglottitis Foreign body aspiration

Correct Answer ( A ) Explanation: Bacterial tracheitis is a rare but life-threatening disease with a peak incidence at 3 -4 years of age. It is caused by severe inflammation of the tracheal epithelium and the production of thick mucopurulent secretions. The lining of the trachea forms a loosely adherent membrane that sloughs into the lumen. It may be confused clinically with croup and epiglottitis. The condition typically begins with a viral prodrome similar to croup (barky cough, stridor, rhinorrhea) that intensifies to the point that the child appears toxic and may exhibit signs of airway obstruction. Features that suggest bacterial tracheitis (rather than croup or epiglottitis) include a viral prodrome followed by acute decompensation; symptoms atypical for croup (high fever, cyanosis, and severe distress); a poor response to traditional croup treatment (aerosolized epinephrine and steroids); and the presence of both inspiratory and expiratory stridor. A soft tissue neck radiograph is nonspecific and may reveal subglottic narrowing, a ragged edge of the usually smooth tracheal column, and a hazy density within the tracheal lumen. The epiglottis and supraglottic structures appear normal. Croup (B) is the most common cause of upper airway distress and obstruction in childhood with a peak incidence at 2 years. It is associated with a barky (seal-like) cough with stridor that is worse with agitation. A patient with moderate croup may be fussy but alert, interactive, and usually comforted by parents. High fever is uncommon. Epiglottitis (C) is rarely seen in vaccinated children. It is also an invasive bacterial disease that causes inflammation and edema of the epiglottis, aryepiglottic folds, and surrounding supraglottic tissues. As these structures become inflamed and distended, they protrude downward and over the glottis opening. A lateral neck radiograph typically shows an enlarged epiglottis ("thumbprint sign"). An upper airway foreign body (D) can cause partial or complete obstruction leading to poor air exchange, ineffective cough, severe distress, and cyanosis. The patient in the clinical scenario is more consistent with an infectious process (URI symptoms, fever) than with foreign body.

Which of the following findings on X-ray is suggestive of colorectal cancer? Apple core sign Bird's beak sign Egg-on-a-string sign String sign

Correct Answer ( A ) Explanation: Colorectal cancer is the third most common type of cancer for both men and women in the United States. It is most commonly seen in adults aged 50 years or older. Risk factors for the development of colorectal cancer include family history of colorectal cancer, inflammatory bowel disease, cigarette smoking, obesity and lack of physical activity. Screening is recommended for all adults starting at age 50 years; earlier depending on risk factors. The majority of patients with colorectal cancer are asymptomatic in the early stages of the disease and are often identified through screening. Later in the disease process, patients with colorectal cancer can present with complications such as large bowel obstruction or perforation. Plain abdominal radiography is a helpful first step in evaluation. Findings include the apple core sign, which occurs when circumferential masses severely constrict the bowel lumen. Establishing the diagnosis requires examination of tissue, which is best accomplished through colonoscopy. Treatment depends on the severity of the disease. Bird's beak sign (B) is an X-ray finding seen with achalasia, which is an esophageal motility disorder. Radiographs of neonates with Transposition of the Great Arteries will have the egg-on-a-string sign (C). The heart is globular or egg-shaped and the mediastinum is narrowed, giving the appearance of a string. Narrowing of the bowel lumen caused by Crohn's disease or pyloric stenosis results in the string sign (D) on X-ray.

You diagnose a patient with myofascial pain syndrome. In an effort to decrease her overall pain levels, which of the following medications is most appropriate to prescribe? Cyclobenzaprine Haloperidol Oxycontin Pregabalin

Correct Answer ( A ) Explanation: Myofascial pain can essentially be defined as the presence of trigger points, focal distinct painful spots located in palpable taut bands of muscles. It is a common musculoskeletal cause of pain, and can be associated with local and distal sensory abnormalities as well as autonomic, sleep and mood disorders. Treatment begins with aggressive physical therapy, which includes specific treatments of myofascial release, ultrasound, spray-and-stretch techniques, posture rebalancing and specific muscle stretch-strengthen exercises. Further treatment options include cognitive-behavior therapy and trigger point injection therapy. Medications also play a role. Muscle relaxants, such as cyclobenzaprine, provide pain relief. NSAIDs, only when used in conjunction with other active treatment, are also beneficial for pain relief. Haloperidol (B) is an antipsychotic not used in the treatment of myofascial pain. Tricyclics such as amitriptyline, however, can be used in treating myofascial pain syndrome. They are used to restore normal sleep patterns, but do not directly decrease the trigger point pain. Pregabalin (D) is cleared for use in fibromyalgia, not myofascial pain syndrome. Oxycontin (C) and other opioid medications are not recommended for myofascial pain syndrome.

A 56-year-old woman with a history of diabetes mellitus type 2 presents to your office with a complaint of bilateral foot pain. She describes the pain as a burning sensation that occurs with rest and improves with activity. Her most recent hemoglobin A1C was 11.3%. Which of the following is the most appropriate initial therapy? Amitriptyline Lamotrigine Oxcarbazepine Oxycodone

Correct Answer ( A ) Explanation: Peripheral neuropathy is a major cause of morbidity in patients with diabetes mellitus. Risk factors for the development of diabetic neuropathy include duration and severity of hyperglycemia, hyperlipidemia, smoking and hypertension. Diabetic neuropathy is a symmetrical sensory polyneuropathy that generally affects the distal lower extremities first. Clinical manifestations of diabetic neuropathy include pain in the feet described as burning or tingling that occurs at rest and improves with activity. Symptoms are sometimes preceded by a recent change in glycemic control. There are three aspects to the treatment of diabetic neuropathy: glycemic control, foot care and pain management. The American Diabetes Association recommends a step-wise management strategy in patients with diabetic neuropathy that begins with excluding other neurologic etiologies and stabilizing glycemic control. Pharmacologic treatment should begin with tricyclic antidepressants such as amitriptyline and then may include anticonvulsants such as gabapentin. The anticonvulsants lamotrigine (B) and oxcarbazepine (C) are not recommended in the treatment of diabetic neuropathy due to being ineffective in managing pain. The role of opiates including oxycodone (D) is controversial in the treatment of diabetic neuropathy due to the risks of abuse, addiction and tolerance. Opiates should not be used as a first-line pharmacologic intervention in patients with diabetic neuropathy.

A six-year-old boy was well until this morning when he awoke with papulovesicular eruptions over his arms, hands, legs, and feet. He refuses to eat or drink. What is the most likely etiology of his symptoms? Coxsackie virus Herpes virus Rickettsia rickettsiae Treponema pallidum

Correct Answer ( A ) Explanation: The above child has classic symptoms and signs of hand-foot-and-mouth disease, a common childhood infection caused by a Coxsackie or enterovirus. The clinical course begins abruptly with a maculopapular or maculopapular exanthem that includes the palms and soles, as well as the extremities. Involvement of the trunk may also be present. The oral enanthem typically involves the mucous membranes of the tongue and buccal mucosa and less commonly the palate or tonsils. The intraoral macules become vesicular and subsequently rupture to form ulcers with grey-yellow bases and circumferential erythema. Intraoral lesions are exquisitely painful, especially upon contact with salty or sour substances. As such, some children will refuses to drink and require admission for intravenous hydration. Preceding symptoms such as malaise or fever are typically absent. Herpes (B) may cause herpetic gingivostomatitis, the manifestation of primary infection with HSV-1. It is most common in children between six months and five years of age. Infection begins with prodromal symptoms such as fever and malaise. Subsequently, the child develops inflammation, bleeding, and friability of the gingiva, along with clusters of intraoral vesicles that progress to painful ulcerations. In contrast to hand-foot-and-mouth disease, the oral enanthem is often preceded by several days of non-specific symptoms. Children may spread herpes to other areas of their body through auto-innoculation with saliva, but the auto-innoculation would not be expected to affect both palms and both soles. Treponema pallidum (D) causes syphilis. While secondary syphilis may cause a variety of appearances of exanthems, including involvement of the palms and soles, mucous membrane involvement is less common. Rickettsia rickettsiae (C) causes Rocky Mountain Spotted Fever (RMSF), which is a tick-borne illness most common in the spring and summer. RMSF begins with several days of non-specific symptoms, including fever, headache, arthralgias, and myalgias. The rash of RMSF typically does not occur until the third or fourth day of illness. The classic rash is initially macular and later petechial, begins on the wrists and ankles, and spreads inward. Palm and sole involvement is characteristic but may not occur until later in the course of infection. Prompt treatment is required to prevent mortality or significant morbidity.

A 26-year-old woman presents to the ED after finding a tick attached to her right flank. She believes it has been there since she went hiking four days prior. On exam, you notice a red annular rash on her right flank with mild central clearing. A urine beta-hCG test is positive. Her last menstrual period was six weeks prior to this visit. What antibiotic prescription should this patient receive? Amoxicillin, 14 days Bactrim, 14 days Doxycycline, 14 days Doxycycline, 7 days

Correct Answer ( A ) Explanation: This patient has a history and physical exam consistent with Lyme disease. Her rash is classic for erythema chronicum migrans, seen in 60%-80% of those with Lyme disease, usually in the first several days after a tick bite. This first stage of Lyme disease needs to be treated before it progresses to more serious symptoms affecting the neurological and cardiovascular systems. She is also pregnant; therefore, she should be treated with a 14-day course of amoxicillin. Bactrim (B) is not one of the antibiotics recommended in the treatment of Lyme disease. Doxycycline (C and D) is the first-line therapy for Lyme disease in the general population. However, it is class D in pregnancy and should not be used in pregnant or lactating women, or for children younger than 8 years old.

Which of the following statements is true regarding appendicitis? An appendicolith is identified in the majority of cases of appendicitis Leukocytosis is seen in the majority of cases Perforation is rare in patients younger than 2 years The presence of an appetite makes the diagnosis unlikely

Correct Answer ( B ) Explanation: Appendicitis is the most common surgical cause of abdominal pain. The highest incidence occurs in patients 10 to 30 years of age, although the highest misdiagnosis rate is in infants and the elderly due to atypical presentations. Leukocytosis (>10,000) occurs in up to 70% of patients diagnosed with appendicitis. The primary inciting event is obstruction of the appendiceal lumen, commonly from an appendicolith, lymphoid hyperplasia, or tumors. Obstruction leads to increased intraluminal pressure and distention of the appendix (visceral pain/periumbilical pain). Increased distention leads to vascular compromise of the appendiceal wall and bacterial invasion leading to localized peritoneal inflammation (somatic pain/RLQ pain). An appendicolith (A) is identified radiographically (most commonly on CT) but only in 10% of patients. The perforation rate (C) in patients under 2 years of age approaches 90% and often occurs as the initial presentation. The presence of an appetite (D) does not rule out appendicitis. A retrocecal appendicitis with limited peritoneal irritation may be associated with minimal gastrointestinal symptoms. Up to 33% of patients diagnosed with appendicitis do not report anorexia on initial presentation.

An 18-year-old man presents after being stabbed in the back. His vital signs are stable and his FAST examination is negative. Physical examination demonstrates a stab wound to the lower back located just to the left of the midline. The patient is unable to move his left leg and reports that his right leg is numb. Which of the following describes his syndrome? Anterior cord syndrome Brown-Sequard syndrome Central cord syndrome Spinal shock

Correct Answer ( B ) Explanation: Brown-Sequard syndrome is a hemisection of the spinal cord most commonly occurring after a penetrating injury. In this syndrome, patients have proprioception and vibratory sensory loss on the ipsilateral side of the injury as well as loss of motor function on the same side. The contralateral side loses pain and temperature sensation since these fibers cross the spinal cord at the level of their nerve roots. The anterior cord syndrome (A) most commonly occurs from a contusion of the spinal cord after a hyperflexion injury. It may also be called by injury or occlusion of the anterior spinal artery. In this syndrome, patients lose motor function and have decreased pain and temperature distal to the lesion. The components of the posterior cord (position, touch and vibratory sensation) are maintained. Central cord syndrome (C) is the most common incomplete spinal cord syndrome seen after hyperextension injuries. The central area of the cord contains components of the pyramidal and spinothalamic tracts. The upper extremities are affected more than the lower extremities because distal parts of the body are located more peripherally in the cord. Spinal shock (D) involves total loss of function distal to the point of injury due to a contusion or concussive injury to the entire spinal cord.

A 63-year-old man with a history of a cholecystectomy and appendectomy presents with abdominal cramping, vomiting and decreased bowel movements. Bowel sounds are decreased. Which of the following is true regarding this patient? Abdominal X-ray can be used to rule out the diagnosis of small bowel obstruction CT scan of the abdomen is highly specific in small bowel obstruction Serum lactate is highly sensitive early on in patients with small bowel obstruction Serum white blood count is always elevated in small bowel obstruction

Correct Answer ( B ) Explanation: CT scan of the abdomen has rapidly become the imaging modality of choice in diagnosing small bowel obstruction (SBO) as it is both highly sensitive and highly specific. Patients with SBO present with crampy abdominal pain with nausea, vomiting, constipation and abdominal distension. The pain can be generalized and often will come in waves. The rapidity of onset of symptoms depends on how proximal the obstruction is. More proximal obstructions will have a more rapid onset and peak of symptoms. CT scan of the abdomen and pelvis is the diagnostic modality of choice as it has a high sensitivity and specificity and can give additional information including location of obstruction, cause of obstruction and alternative diagnoses. Abdominal X-ray (A) has poor sensitivity and specificity. There are no lab tests (C, D) that aid in diagnosis.

A 66-year-old man with Type 2 diabetes presents complaining of right ear pain and drainage for the past 3 weeks. He states the pain is deep in the ear and that the drainage is greenish and foul-smelling, and has increased over the past 3 weeks. He denies cough, congestion, fever, or placing anything in his ear. On physical examination, the patient is afebrile. Otoscopic examination reveals a markedly edematous right ear canal draining purulent, green discharge. The tympanic membrane is unable to be visualized. Examination of the left ear is unremarkable. Which of the following is the most likely diagnosis? Acute mastoiditis Auricular cellulitis Chronic otitis externa Malignant otitis externa

Correct Answer ( D ) Explanation: Malignant (necrotizing) otitis externa is an invasive infection of the external auditory canal and skull base. It is a rare complication of persistent otitis externa. Diabetic and immunocompromised patients are particularly at risk. It usually presents with persistent foul discharge, granulations in the ear canal, deep otalgia, and in advanced cases, cranial nerve palsies. Physical examination findings include an edematous ear canal with the presence of granulation tissue resulting in difficulty visualizing the tympanic membrane during otoscopic exam. Diagnosis is confirmed by the demonstration of osseous erosion on CT and radionuclide scanning. Accurate diagnosis is needed as this is a potentially life threatening infection. Acute mastoiditis (A) presents with pain, tenderness, and swelling over the mastoid. This can accompany an untreated acute otitis media, but is rare with prompt antibiotic treatment. Auricular cellulitis (B) usually presents with swelling, erythema, and tenderness of the ear, primarily involving the lobule of the ear. There would not be any associated otorrhea. Chronic otitis externa (C) presents commonly with pruritus and rarely is associated with ear pain. It is usually caused by irritation from repeated minor trauma to the ear canal or drainage from a chronic middle ear infection.

A 24-year-old woman presents complaining of "bumps" in her genital area for the past several weeks that seem to be getting larger. She is sexually active. Physical exam reveals small, non-tender, flesh-colored, papillary growths on the vulva as shown. What is the most likely diagnosis? Acrochordon Condyloma acuminata Molluscum contagiosum Sebaceous cyst

Correct Answer ( B ) Explanation: Condyloma acuminata, or genital warts, are caused by an infection with the human papilloma virus (HPV). Mainly types 6 and 11 present in this manner. It is a sexually transmitted infection (STI) that can be prevented with vaccination. Symptoms include white to flesh-colored, exophytic or papillomatous growths in the genital area. In women, this can affect the vagina, cervix, vulva, oropharynx, perineum, and perianal areas. HPV can be spread via sexual contact or from mother to newborn during passage through the birth canal. Treatment includes trichloroacetic acid solution, podophyllin, cryosurgery, surgical excision, or imiquimod cream application. Acrochordon (A) is a flesh-colored growth commonly known as a skin tag. Molluscum contagiosum (C) are dome-shaped with an umbilicated center. Molluscum contagiosum can also be transmitted sexually and is due to a DNA poxvirus. Sebaceous cyst (D) is a general term that describes both epidermoid and pilar cysts. These cysts contain keratin and not sebum and neither originate from the sebaceous gland. True sebaceous cysts are rare and known as steatocystomas. Sebaceous cysts are mobile masses that consist of fibrous tissue and fluids, sometimes it resembles cottage cheese or can be purulent serosanguinous fluid.

A postmenopausal woman presents to the clinic worried about her and her husband's sexual health. As of late, sexual intercourse has been very uncomfortable for the both of them. She thinks it has to do with her recurrent urinary tract infections, however, you explain that as a woman ages, vaginal atrophy occurs due to a significant decrease in which of the following hormones? Androstenedione Estrogen Progesterone Testosterone

Correct Answer ( B ) Explanation: During menopause, the ovary becomes less sensitive to circulating follicle-stimulating hormone, and as such, produces less estrogen. Estrogen is largely responsible for the health of the female urogenital epithelium, acting on receptors located in the vagina, vulva, urethra and urinary bladder. It maintains the proper amount of collagen, mucopolysaccharides and hyaluronic acid in the epithelium of these organs. As estrogen levels decrease significantly as a woman passes thru menopause, urogenital epithelial changes occur, and as a result, vaginal dryness and irritation can occur. The subsequent decrease in vaginal lubrication can cause painful sexual intercourse. Also, thinning and irritation of the urinary epithelium can lead to recurrent urinary tract infections. Decreased androstenedione (A), and other androgens like testosterone (D), have been implicated in female pattern hair loss, or androgenic alopecia, in postmenopausal women. Progesterone (C) levels also decrease during menopause, however, this female hormone is responsible for uterine ripening, a process necessary to allow proper implantation of a fertilized ovum into the uterine wall.

Which of the following is a mainstay for the treatment of Kawasaki disease during the acute febrile phase? Aspirin - low dose Intravenous immunoglobulin (IVIG) Steroids Warfarin

Correct Answer ( B ) Explanation: Intravenous immunoglobulin (IVIG), as well as high dose aspirin, form the two components of the treatment of Kawasaki disease during the acute febrile phase of the disease. IVIG involves an infusion of 2 g/kg over 10 to 12 hours. High dose aspirin (80-100mg/kg/day divided into 6 hour dosing regimens) is continued until the child is afebrile for 48 to 72 hours or longer. Low dose aspirin (A) (3 to 5 mg/kg/day) is then initiated and continued until the patient shows no coronary changes by 6 to 8 weeks. The benefit of steroids (C) is controversial and current guidelines recommend restricting steroid treatment to children in whom >2 infusions of IVIG have been ineffective in alleviating fever and acute inflammation. When a patient with Kawasaki disease develops coronary artery disease, treatment may involve antiplatelet therapies (aspirin, with or without dipyridamole or clopidogrel), anticoagulant therapy with warfarin (D) or low-molecular-weight heparin, or a combination of anticoagulant and antiplatelet therapy (usually warfarin plus aspirin). No prospective data exist to guide clinicians in regards to the optimal regimen.

Which of the following is most likely to be seen on the physical exam of a patient with hypothyroidism? Alopecia areata Delayed relaxation of deep tendon reflexes Lid lag Onycolysis

Correct Answer ( B ) Explanation: Lack of thyroid hormone causes a general slowing of metabolic processes. Clinical signs and symptoms reflect this slowing. Laboratory testing is the primary means of diagnosing hypothyroidism because clinical findings are non-specific. However, clinical manifestations of thyroid deficiency can provide clinicians with the clinical suspicion needed to order such testing. Delayed relaxation of deep tendon reflexes, hair thinning, periorbital edema, hypertension, weight gain, and shortness of breath are all physical findings commonly seen in patients with hypothyroidism. Symptoms generally begin to resolve approximately 2-3 weeks after the initiation of thyroid hormone replacement. Alopecia areata (A) is sometimes seen in patients with hypothyroidism after treatment of Graves' disease, but is more commonly seen in patients with hyperthyroidism. Lid lag (C) and onycolysis (D) are physical findings seen with hyperthyroidism.

An 8-year-old girl presents to the emergency department with a one-day history of rash. The rash is erythematous, macular, blanching, and covers her trunk and extremities. It is not painful or pruritic, and there are no vesicles, bullae or mucosal lesions noted. She has bilateral lymphadenopathy in the posterior cervical chains with splenomegaly on palpation of her abdomen. The rest of her physical exam is normal. Her pediatrician started an antibiotic last week for pharyngitis but she has not improved. What is the most likely etiology of her rash? DRESS Syndrome Mononucleosis Stevens-Johnson Syndrome Type I Hypersensitivity reaction

Correct Answer ( B ) Explanation: Mononucleosis is caused by the Epstein Barr virus (EBV) and presents clinically with fatigue, fever, pharyngitis, lymphadenopathy, and splenomegaly. Mononucleosis is associated with a faint, widespread, non-pruritic rash, which lasts for about a week. It is characterized by patches that may contain small bumpy red spots. It thought to be directly due to the virus. This rash often appears on the trunk and upper arms first, and a few days later extends to involve the face and forearms. This is in contrast to a more intense pruritic maculopapular or morbilliform which appears on extensor surfaces and pressure points 7 to 10 days after treatment with beta-lactam antibiotics such as ampicillin, amoxicillin and cephalosporins. This rash indicates a hypersensitivity reaction to the antibiotic. It is not a true allergy and does not occur if the antibiotic is given later on in the absence of EBV infection.The peak incidence of infection with EBV occurs in adolescence. The diagnosis can be made with EBV IgM and IgG or EBV PCR. Rare but life-threatening complications of mononucleosis include splenic rupture; therefore patients with mononucleosis should be instructed to avoid contact sports until their illness has resolved. DRESS Syndrome (A), or drug-induced hypersensitivity syndrome, typically presents with fever, severe exanthematous rash that can be exfoliative, generalized lymphadenopathy, and hematologic abnormalities. It is usually associated with anticonvulsants, as well as minocycline, sulfonamides, nitrofurantoin, and allopurinol. This syndrome can be life threatening due to organ damage such as carditis, hepatitis, and nephritis. Eosinophilia is present in up to 50% of cases. Mucosal lesions are rarely seen in this disorder, differentiating it from Stevens-Johnson syndrome or toxic epidermal necrolysis. Stevens-Johnson Syndrome (C) (SJS) is a severe mucocutaneous reaction that presents after drug exposure. It is on a continuum with erythema multiforme and toxic epidermal necrolysis. SJS is seen most often with anti-convulsants and sulfa drugs and is characterized by fever, malaise, arthralgias, and a rash. The rash is tender, erythematous coalescing macules with central purpura that may evolve into vesicles or bullae. Type 1 Hypersensitivity reactions (D) occur immediately after exposure to the antigen, rather than after 1 week of exposure. These reactions can present with urticaria, bronchospasm, or anaphylaxis.

A patient with dyspnea and angina fails medication management of his symptoms with beta-blockers, ACE-inhibitors and calcium channel blockers. He undergoes complete cardiac evaluation which uncovers the presence of nonobstructive, end-stage hypertrophic cardiomyopathy. Which of the following is the most appropriate treatment at this point in time? Aggressive diuresis Cardiac transplantation Implantable intracardiac pacing Surgical myectomy

Correct Answer ( B ) Explanation: The management of hypertrophic cardiomyopathy should follow heart failure treatment guidelines. This includes a careful use of diuretics, as many patients with HCM require higher filling pressures to maintain cardiac function. This is especially true if edema is not a main finding. Negative inotropes and negative chronotropes are also recommended, and include beta-blockers and calcium channel blockers. If pharmacotherapy is unsuccessful, further treatment is dictated by whether the hypertrophic cardiomyopathy is obstructive or nonobstructive. If obstructive physiology is detected, surgical options may be required, and include cardiac pacing, surgical myectomy or septal ablation. If nonobstructive physiology is found, cardiac transplant is usually the only viable solution. Diuresis (A) is cautioned in those with hypertrophic cardiomyopathy. Acute, aggressive diuresis may likely push the patient into further decompensation. Pacing (C) and myectomy (D) may be viable treatment options for obstructive HCM but not for nonobstructive HCM.

A six-month-old infant boy presents to his pediatrician's office for a follow-up visit. His parents are concerned about his immunizations because an otherwise healthy and normal older brother died of varicella complications at a year of age. They are aware of the fact that varicella virus is highly infectious. They request your advice about their son's varicella immunization schedule. Which one of the following recommendations is most appropriate? Administer the killed varicella vaccine at 12 months and two years of age Administer the live varicella vaccine at 12 months and four years of age Evaluate this infant for immune deficiency and administer a dose of varicella-zoster immune globulin (VZIG) prior to any further immunizations Immunize this six-month-old infant with his first dose of live varicella vaccine at this visit and the second dose at nine months of age

Correct Answer ( B ) Explanation: This child should be immunized following the Advisory Committee on Immunization Practices (ACIP) recommendations. The specific recommendation is to administer the live varicella vaccine at 12 to 15 months and four to six years of age. The history of a fatal case of varicella in the older brother at a year of age, although unusual, does not imply any immune problem in the younger brother. Carefully done prospective studies of deaths from varicella done prior to the availability of the live vaccine suggested a much larger morbidity and mortality than previously appreciated and supported the use of the varicella vaccine. Administration of the killed varicella vaccine at 12 months and two years of age (A) is not possible because there is no commercially available or recommended killed varicella vaccine. Evaluation this infant for immune deficiency and administration of a dose of VZIG prior to any further immunization (C) is also inappropriate. The rationale for both the use of a killed vaccine and immune evaluation might be the anticipation of the older brother having had a cellular immune deficiency, which is unlikely. Similarly, the administration of VZIG, implies an immune-deficient host. However, it is only indicated in immune deficient hosts who have had a recent probable exposure to an individual with active varicella. VZIG may be difficult to obtain. Immunization of this infant at six and nine months of age (D) is neither indicated nor recommended.

A 3-year-old boy presents with severe vomiting and diarrhea. The exam reveals sunken eyes, skin tenting and a capillary refill of 3 seconds. He weighs 13 kg. He was 15 kg at his last well visit. Using the Holliday-Segar method, what is his maintenance fluid rate per hour? 260 ml/hr 39 ml/hr 47.9 ml/hr 83.3 ml/hr

Correct Answer ( C ) Explanation: 47.9 ml/hr is calculated utilizing the Holliday-Segar method of 100 ml/kg/day for the first 10 kg, 50 ml/kg/day for the next 10 kg and 20 ml/kg/day for each additional kilogram. This total is then divided by 24 hours. (100 ml/kg/day x 10 kg) + (50 ml/kg/day x 3kg) = 1150 ml/kg/day/24hr = 47.9ml/hr This method is sometimes shortened to the 4-2-1 method of 4 ml/kg/hr for the first 10 kg, 2 ml/kg/hr for the next 10 kg and 1 ml/kg/hr for each additional kilogram. (4 ml/hr x 10 kg) + (2 ml/hr x 3 kg) = 46 ml/hr If isonatremic, sodium and potassium maintenance would need to be calculated and added into this maintenance fluid for a proper order to be written. 39.0 ml/hr (B) is similar to the Parkland formula of fluid replacement that is utilized with burn patients. This formula for children is 3 ml x weight (kg) x % body surface area burned + maintenance. For adults, the formula is 4 ml x weight (kg) x % body surface area + maintenance. The fluid is replaced in increments with the one half given over the first 8 hours and the remainder given over the next 16 hours. Lactated ringers are the fluid of choice for burns. 83.3 (D) is this patient's fluid deficit. This is calculated as Fluid deficit (L) = pre-illness weight (kg) - illness weight (kg). In this case, 15kg - 13kg = 2L. The 2 L fluid deficit is divided by 24 hours to yield 83.3 ml/hr. If the pre-illness weight is unavailable, clinical observations may be utilized to determine the percent dehydration and calculated as percent (%) dehydrated x weight (kg) = Liters down. 260ml/hr (A) is the emergency bolus delivered and is done as quickly as possible. Boluses are delivered at the rate of 20 ml/kg unless a cardiac condition is present or suspected.

Which of the following is the greatest risk factor for an ectopic pregnancy? Multiple sexual partners Previous abdominal surgery Previous ectopic pregnancy Previous pelvic inflammatory disease

Correct Answer ( C ) Explanation: An ectopic pregnancy is when implantation of the gestational sac occurs outside of the uterus. The most common location for an ectopic pregnancy is the fallopian tubes. It can also occur in the interstitial or cornual portion of the uterus (2%), intraabdominally (1.5%), on the ovary (0.1%) and or within the cervix (0.1%). A history of a previous ectopic pregnancy is regarded as the greatest risk factor for an ectopic pregnancy with an odds ratio of 8.3. Other high-risk features include previous tubal surgery and in utero diethylstilbestrol (DES) exposure. Multiple sexual partners (A) and previous abdominal surgery (B) increase the risk of ectopic pregnancy modestly with an odds ratio of 0.9-3.8. Previous pelvic inflammatory disease (D) is a strong risk factor for an ectopic pregnancy with an odds ratio of 2.1 and is associated with up to 50% of ectopic pregnancies.

Which of the following statements is true concerning aspiration pneumonitis? Antibiotics should be administered early in the course Corticosteroid administration decreases the incidence of associated lung injury It is caused by an inflammatory chemical injury The ABG is diagnostic

Correct Answer ( C ) Explanation: Aspiration pneumonitis is an inflammatory chemical injury of the tracheobronchial tree and pulmonary parenchyma produced from the inhalation of regurgitated gastric contents. Aspiration pneumonitis can lead to aspiration pneumonia due to the breakdown of the pulmonary defense mechanisms caused by the chemical irritation. Aspiration pneumonitis is usually associated with depressed level of consciousness that allows for regurgitation of gastric contents and inhibits the protective upper airway reflexes from preventing aspiration. The classic patient has a depressed mental status due to recreational or therapeutic drugs and regurgitates a large volume of gastric contents. The elderly and those patients with a history of stroke are also at increased risk for aspiration pneumonitis. Clinically, patients may have minor symptoms (nonproductive cough and tachypnea). With larger aspirations, patients may develop tracheobronchitis with bronchospasm, bloody or frothy sputum, and respiratory distress. The chest radiograph usually shows unilateral focal or patchy consolidations in the dependent lung segments. Prophylactic administration of antibiotics (A) is not recommended and may lead to increased resistance. There is no evidence that corticosteroid (B) administration decreases associated lung injury. Laboratory values are of little diagnostic value. The ABG (D) may reflect hypoxia or hypoventilation, but this may be secondary to the patient's underlying lung disease.

An 18-year-old woman with a history of frequent visits to the ED presents with multiple superficial lacerations to her left wrist. She is agitated and states she is suicidal. She is yelling at the nurses and demands to be seen only by the psychiatrist. Which of the following is the most appropriate next step in management? Administer 5 mg of intramuscular haloperidol Consult the psychiatry service Engage in conversation and discuss her symptoms and validate her distress Place her alone in a private room and wait for her to calm down

Correct Answer ( C ) Explanation: Borderline personality disorder (BPD) is a common emergency psychiatric presentation. BPD is characterized by unstable personal relationships, unstable self-image, and inappropriate behaviors. The disorder may include chronic feelings of emptiness, which may be misdiagnosed as depression, or lability of mood, which may be mistaken for mania or hypomania. Borderline patients typically live lives of crisis and constant conflict. Patients with BPD usually present to the ED after deliberate self-injury or suicidal attempts. Recurrent suicidal threats or acts and self-injury with a combination of strong preoccupation with expected rejection and abandonment are the strongest indicators. These patients feel they need to be connected to someone who they believe really cares. Therefore, the first step is to engage the patient in conversation and discuss his or her symptoms and validate his or her distress. Chemical sedation (A) is an option if the patient becomes physically threatening to herself or others. However, the patient should first be engaged in behavior modification. The psychiatrist (B) should ultimately be consulted, but after the patient has been stabilized and a complete medical exam is performed. The patient should not be left alone in a room (D) as she is at risk for harming herself. Patients with borderline personality disorder are seeking validation from others and therefore will continue to be disruptive until they can be engaged in conversation.

During an intake history, a patient describes her husband as a violent, angry man. You suspect domestic violence of a physical nature. You counsel the woman that serious injury or death most commonly occurs during which of the following situations? After an argument about financial issues After an argument about sex-life issues When she tries to leave the husband When the husband tries to apologize for his actions

Correct Answer ( C ) Explanation: Domestic abuse, also known as domestic violence, spousal abuse or intimate partner violence, is a behavior pattern in which the abuser displays any of the following against a partner: aggression, assault, control, intimidation, stalking, sexual abuse, mental or psychological abuse or economic deprivation. Three phases of abuse have been described. The tension building phase is characterized by poor communication, tension and fear. This is followed by a violent phase or episode. The honeymoon phase then follows, and is characterized by the abuser offering apology, sympathy, affection and remorse. In those abused, especially if they continue to live with the abuser, a high incidence of psychological disorders exists, and includes, stress, fear, generalized anxiety disorder, panic disorder, depression, suicide and post-traumatic stress disorder. One study shows 60% of victims meet the diagnostic criteria for depression, either during or after termination of the relationship. Physical abuse can occur during a relationship. However, assaults occur more commonly near the end of relationships. Up to 75% of domestic assaults occur during an attempt to report abuse or leave an abusive relationship. Common occurrences in domestic violence marriages include arguments about financial issues (A), arguments about sex-life issues (B) and apologizes from the husband after his actions (D). They are, however, not the most common situations that lead to significant physical assault or death.

Which of the following has the highest relative proportion of rabies in the United States? Bats Dogs Raccoons Skunks

Correct Answer ( C ) Explanation: Since the 1950s, rabies in U.S. domestic animals has decreased significantly, but the population of rabid wild animals has actually increased. The primary reservoirs are raccoons (38%), bats (24%), skunks (22%), and foxes (6%). Although raccoons have the highest proportion of rabies in the U.S., most cases of human rabies in the have been attributed to bats. Bats (A) make up 24% of rabies in wild animals. Transmission from bats appears to occur from seemingly minor or unrecognized bites. Therefore, any exposure to a bat should prompt strong consideration to initiate rabies prophylaxis. Dogs (B) make up less than 5% of rabies cases. Most cases are seen along the United States-Mexico border in unvaccinated dogs. Skunks (D) make up a large proportion of rabies in the U.S. A bite from a skunk should prompt rabies prophylaxis.

After taking a normal history, performing a normal physical exam, and obtaining a normal 12-lead ECG, which of the following is the most common cause of syncope? Dysrhythmia Hypovolemia Unknown Vasovagal

Correct Answer ( C ) Explanation: Syncope is defined as a sudden temporary loss of consciousness with the inability to maintain postural tone. It is a symptom with a wide variety of life-threatening causes. Risk factors for syncope include cerebrovascular disease, cardiac manifestations, and hypertension. Despite extensive workups, no cause for syncope is found in approximately 50% of cases. When a diagnosis is made, most causes of syncope are benign and have favorable outcomes. Patients with preexisting cardiovascular disease and syncope from any cause are at the greatest short- and long-term risk of mortality. Syncope from cardiovascular causes is associated with increased mortality, whereas syncope due to neurocardiogenic, orthostatic, and medication-related syncope is not associated with increased mortality. Cardiac dysrhythmias (A) are the most common cause of cardiac syncope. Brady dysrhythmias such as heart block can lead to syncope in addition to tachydysrhythmias such as ventricular tachycardia and supraventricular tachycardia. Orthostatic hypotension, dehydration, and hypovolemia (B) can cause syncope. The most susceptible individuals are elderly patients taking blood pressure medications or diuretics. Syncope can also be the presenting symptom secondary to hemorrhage such as seen with an ectopic pregnancy. Vasovagal (D), or situational syncope, is a common cause of syncope. Multiple factors contribute to these attacks. There is typically a relatively low blood volume. Various circumstances can then lead to the syncopal event. If there is underlying fear or anxiety, the vasomotor center increases heart rate in order to increase cardiac output. However, due to a relatively low blood volume, the heart is unable to meet the requirement. The high sympathetic activity is always modulated by vagal outflow, in these cases leading to excessive slowing of heart rate. This leads to a paradoxical vagal response that causes syncope. The tilt-table test typically evokes the attack.

A 12-year-old boy has a cognitive disability and multiple café-au-lait macules. Which additional finding is most likely on examination? Annular, hypopigmented macules Facial angiofibromas Neurofibromas Periungal fibromas

Correct Answer ( C ) Explanation: The boy has both café-au-lait macules and cognitive disability, raising concern for neurofibromatosis type 1, or von Recklinghausen's disease. Neurofibromatosis is a neurocutaneous disorder caused by a mutation on NF1 on chromosome 17. Cutaneous findings of neurofibromatosis type 1 include café-au-lait macules, axillary and inguinal freckling, Lisch nodules, and neurofibromas. Neurofibromas are peripheral nerve sheath tumors that may be focal and cutaneous or plexiform. Cutaneous neurofibromas are small, sessile or pedunculated lesions that move with the skin. They are non-tender and occur primarily on the trunk. Cutaneous neurofibromas most commonly develop during adolescence, although subtle lesions may be noted earlier in angled light. The number of neurofibromas on an affected individual may range from a few to thousands. Plexiform neurofibromas are present in half of patients with neurofibromatosis and cause a very different clinical picture. They may be superficial, deep, or both. The entangled nerves can become large, complex, and often disfiguring due to associated overgrowth of surrounding tissues. Deep plexiform neurofibromas may compress internal structures, including the airway and spinal cord. Individuals with neurofibromatosis are also at increased risk of bony abnormalities, including bony dysplasia, pseudoarthroses, scoliosis, osteoporosis, and short stature. Additionally, affected individuals are predisposed to a variety of tumors, including optic nerve gliomas and soft tissue sarcomas such as rhabdomyosarcoma, malignant peripheral nerve sheath tumors, and glomus tumors. Annular, hypopigmented macules (A) in a patient with neurological abnormalities are most consistent with the ash-leaf spots of tuberous sclerosis. Like neurofibromatosis, tuberous sclerosis is a neurocutaneous disorder and so should be considered in any child with both unusual cutaneous findings and neurodevelopmental abnormalities. Facial angiofibromas (B), also known as adenoma sebaceum, and periungal fibromas (D) are also features of tuberous sclerosis. Both tend to develop during adolescence, but ash-leaf spots develop earlier in childhood.

A 64-year-old man complains of pain and paresthesias in his right hand intermittently for several weeks. He works in a factory putting together electronics. On exam, he has decreased sensation of his right 1st through 4th digits and an atrophied thenar eminence. What test in the ED will help diagnose his condition? CT scan of the head Electromyelography (EMG) Percuss the right volar wrist Urine drug screen

Correct Answer ( C ) Explanation: This patient has median mononeuropathy, also known as carpal tunnel syndrome, a compression neuropathy of the median nerve as it traverses under the flexor retinaculum at the wrist. The median nerve provides sensation primarily to the palmar aspect of the 1st, 2nd, 3rd, and radial side of the 4th. When it is compressed, the patient experiences pain, paresthesias, and numbness in that distribution. The Tinel's test is performed by lightly tapping the volar surface of the wrist over the median nerve. This should elicit a sensation of tingling or pins and needles in the distribution of the median nerve. Carpal tunnel syndrome is first treated with wrist splinting and initiation of a more ergonomic work environment. NSAIDs may also be helpful. If symptoms do not improve, the patient should be referred to a hand specialist who may elect to perform a carpal tunnel release procedure. CT scan of the head (A) would be useful if there is suspicion that this patient's symptoms are from a central process such as an acute stroke. However, his numbness is in a peripheral nerve distribution and not dermatomal. An EMG (B) is used most commonly by neurologists to confirm damage to peripheral nerves. It may be used for carpal tunnel syndrome if the symptoms do not resolve with conservative management. A variety of heavy metals are associated with a peripheral neuropathy (lead, mercury), but these require special serologic testing, not a urine drug screen (D). The patient is at risk for occupational exposures because he works in a factory; however, most heavy metal poisonings are associated with other symptoms.

A 17-year old man presents to his healthcare provider for a two-week history of ear pain and pruritus. On further questioning, he states that also has ear fullness and has been having trouble hearing. He is a student and is on a swim team at school. He has had these episodes in the past but not to this extent. On physical exam, his tympanic membrane is difficult to visualize but it is mobile without any inflammation. There is pain on palpation of the tragus. Pain is also elicited upon traction of the pinna. Which of the following treatments should be prescribed to this patient? Amoxicillin Amoxicillin/clavulanate Hydrocortisone/neomycin/polymyxin B Tacrolimus cream

Correct Answer ( C ) Explanation: This patient has otitis externa based on his clinical history and physical symptoms. It is important to differentiate otitis externa from otitis media. Otitis externa is either caused by a bacterial or fungal infection and a history of swimming or using improper cleaning techniques which cause minor trauma are common risk factors for otitis externa. Although there can be a fungal etiology, acute otitis externa is primarily caused by bacterial infection. Acute otitis externa presents with the rapid onset of ear canal inflammation, resulting in otalgia, itching, canal edema, canal erythema, and otorrhea. On physical exam, the tympanic membrane can be difficult to visualize however it is not bulging and is mobile, unlike in otitis media. Key physical features of otitis externa include pain upon palpation of the tragus and upon traction of the pinna. Although this is a superficial infection, a combination of steroid and antibiotic drops have shown to be the most efficacious in the treatment of otitis externa. Hydrocortisone/neomycin/polymyxin B drops are the first-line treatment agent for otitis externa. Amoxicillin (A) and amoxiciilin/clavulanate (B) are used to treat otitis media not otitis externa. Tacrolimus cream (D) is used for resistant otitis externa that is resistant to antibiotic or antifungal medications.

Which of the following is the hallmark characteristic of mumps? Cough, coryza, and conjunctivitis Epididymoorchitis Maculopapular rash Nonsuppurative parotid swelling

Correct Answer ( D ) Explanation: Mumps is a viral illness characterized by fever, swelling, and tenderness of the salivary glands, with the parotid gland most commonly affected. The disease is seen most commonly in the winter and spring months and is communicable 7-10 days after the onset of parotitis. Nonsuppurative parotid swelling is the hallmark of mumps. The swelling can be unilateral or bilateral and is sometimes associated with trismus. Less commonly, patients experience epididymoorchitis, which also can be unilateral or bilateral, and meningitis. The CSF in these cases usually demonstrates a lymphocyte pleocytosis and low glucose. Rare complications include transverse myelitis, Guillain-Barré syndrome, pancreatitis, myocarditis, and deafness. Treatment is supportive. Cough, coryza, and conjunctivitis (A) are associated with measles (rubeola) and precede the characteristic maculopapular rash by 2-4 days. The classic rash of measles is maculopapular (C). The rash starts along the hairline, then proceeds downwards until it reaches the feet. The rash begins on the third day of the illness and peaks at the height of fever and constitutional symptoms (between days 4 and 6). The rash undergoes brown staining that does not blanch after 3 to 4 days and represents capillary hemorrhage. The rash starts to resolving in order of appearance between days 7 and 9. Epididymoorchitis (B) is associated with mumps but is much less common than nonsuppurative parotid swelling is.

A 26-year-old sexually active woman presents to the clinic with several days of vulvovaginal discomfort and pruritus. A pelvic exam shows copious frothy green vaginal discharge, inflamed vaginal walls, and a cervix with punctate hemorrhages. This physical exam is most consistent with which of the following causes of vaginitis? Atrophic vaginitis Bacterial vaginosis Trichomonas vaginalis Vulvovaginal Candidiasis

Correct Answer ( C ) Explanation: Trichomonas vaginalis is a common infectious cause of vulvovaginal discomfort in women that will likely present with the above symptoms. Though Trichomonas vaginalis is often harbored asymptomatically for periods of 5 days to 4 weeks, patients who are diagnosed while symptomatic typically present with vulvovaginal pruritus, dyspareunia, dysuria, or pelvic discomfort. A physical exam will show a classic copious, yellow or green, frothy vaginal discharge. Examining the cervix may show punctate hemorrhages, referred to as a "strawberry cervix." The diagnostic test of choice is a wet mount of the vaginal discharge, which shows mobile trichomonads. The first line treatment for Trichomonas vaginalis is oral metronidazole. All infected sexual partners should also be treated, even if asymptomatic, to prevent the development of symptomatic disease and reduce spread to future partners. Atrophic vaginitis (A) is generally a condition of menopausal women in low-estrogen states that causes thinning of the vaginal mucosa and decreased vaginal lubrication leading to vaginal discomfort and dyspareunia. A pelvic exam shows pale mucosa and a decrease in size of the introitus and cervix. Bacterial vaginosis (B) is a non-sexually-transmitted disease caused by an overgrowth of Gardnerella and other anaerobes. The hallmarks of this disease are a heavy, malodorous grey discharge without presence of vaginitis, increased vaginal pH, and "clue cells" (epithelial cells with borders obscured by the presence of excessive bacteria) visualized on a wet mount. There may also be a notable amine, or "fishy", odor with the addition of potassium hydroxide to a drop of the vaginal discharge. Vulvovaginal Candidiasis (D) is an overgrowth of the yeast Candida which cause vaginal burning and pruritus. The physical exam should show vaginitis, erythema, and a white, curd-like vaginal discharge with no apparent odor. A wet mount with potassium hydroxide added will show spores and hyphae.

A 57-year-old woman is brought by ambulance to the emergency room. Three hours ago she began having a headache with nausea and noticed a loss of sensation on most of the right side of her body. A non-contrast computed tomography scan of her head shows a 2 cm area of enhancement in the brain parenchyma. Which of the following is the most likely location of the patient's lesion? Cerebellum Frontal lobe Temporal lobe Thalamus

Correct Answer ( D ) Explanation: A thalamic hemorrhage classically presents with contralateral hemisensory loss. Other symptoms such as hemiparesis and ocular symptoms can also occur when the internal capsule and other nuclei are involved. Symptoms are acute and progress rapidly. Most patients present with headache and often nausea as well. Non-contrast CT scan is the diagnostic test of choice for brain hemorrhage as it can be done quickly and new blood will enhance on the scan (older blood will appear dark). Prognosis is in large part related to the amount of bleeding. Cerebellar (A) hemorrhages most often present with symptoms of increased intracranial pressure due to compression of the fourth ventricle (lethargy, nausea, vomiting). Compression of the brainstem can also occur leading to comatose before seeing hemiparesis. Other lesions of the cerebellum can cause various forms of ataxia. Frontal lobe (B) lesions can cause a contralateral palsy due to the primary and pre-motor cortex in the frontal lobe but the primary sensory cortex is in the parietal lobe. Broca's area is also in the frontal lobe and damage would lead to an expressive aphasia (good understanding but difficulty producing comprehendible speech). A lesion in the dominant temporal lobe (C) could lead to a Wernicke's aphasia (receptive aphasia) where the patient speaks clearly but has trouble understanding.

Which of the following drugs must be prescribed in a patient with cervicitis in whom bacterial vaginosis is also suspected? Azithromycin Ceftriaxone Doxycycline Metronidazole

Correct Answer ( D ) Explanation: Bacterial vaginosis (BV) is a syndrome caused by polymicrobial overgrowth and requires treatment with either metronidazole or clindamycin. The typical species involved include anaerobic species (Mobiluncus, Bacteroides, Prevotella), Gardnerella vaginalis and Mycoplasma hominis. BV is the most common cause of vaginal discharge but may be asymptomatic in up to half of women. It typically manifests with malodorous vaginal discharge. This discharge is thin and white in consistency. Diagnosis of BV is based on the Amsel criteria: 1) thin, white, homogenous discharge, 2) presence of clue cells (epithelial cells stippled with bacteria), 3) vaginal pH > 4.5 and 4) fishy odor of discharge with addition of 10% KOH (whiff test). Treatment for BV is with metronidazole 500 mg PO BID or clindamycin 300 mg PO BID for 7 days. Alternatively, metronidazole and clindamycin can be prescribed as a vaginal suppository. Unlike with other sexually transmitted infections, treatment of the partner does not reduce the response rate or rate of recurrence. Azithromycin (A) is often used to treat Chlamydia trachomatis infections in cervicitis or urethritis. Ceftriaxone (B) is a 3rd generation cephalosporin that is active against Neisseria gonorrhoea. Doxycycline (C) treats C. trachomatis infections.

A 10-month-old boy is being evaluated for bilious vomiting, diarrhea, and rectal bleeding. He appears to be in severe distress and is continuously crying. Which of the following should be used to both diagnose and treat this patient's condition? Abdominal CT scan Abdominal ultrasound Abdominal X-ray Contrast enema

Correct Answer ( D ) Explanation: Based on the constellation of findings, this patient most likely has intussusception. Intussusception occurs when one portion of the intestine invaginates into another portion. The most common form is intussusception of the terminal ileum into the right colon (ileocolic intussusception). Intussusception is the most common abdominal emergency in early childhood. Patients with a typical presentation (eg, infant or toddler with sudden onset of intermittent severe abdominal pain with or without rectal bleeding), can proceed directly to nonoperative reduction using a contrast enema (using either barium or air). In these cases, the procedure is both diagnostic and therapeutic. In patients with an atypical presentation of intussusception, ultrasonography is the method of choice to detect intussusception in many institutions. The classic ultrasound image of intussusception is a "target sign" which represents concentric circles of bowel. An abdominal CT (A) can show the intussusception, however, it can be time-consuming in children who may require sedation. Ultrasonography (B) can be used in patients with an atypical presentation of intussusception, however, it is not therapeutic when compared to contrast enema. Although an abdominal X-ray (C) is often used in the evaluation of acute abdominal conditions, it is less sensitive and less specific than ultrasonography for the diagnosis of intussusception.

A 63-year-old man with a 40-year history of alcohol and tobacco abuse presents with solid food dysphagia. The patient has also had a 21 lb weight loss over the past 7 months. Which of the following studies should be performed for a definitive diagnosis? Barium esophagram Chest X-ray CT Scan Endoscopy

Correct Answer ( D ) Explanation: Endoscopy with biopsy establishes the diagnosis of esophageal carcinoma with a high degree of reliability when biopsy is included as part of the procedure. Esophageal cancer commonly develops in persons between 50 and 70 years of age. The overall incidence ratio of men to women is 3:1. There are two histological types: Squamous cell carcinoma and Adenocarcinoma. Chronic alcohol and tobacco use are strongly associated with an increased risk of squamous cell carcinoma. The majority of adenocarcinomas develop as a complication of Barrett's metaplasia due to chronic gastrostroesophageal reflux (GERD). Solid food dysphagia and significant weight loss are common symptoms of advanced, incurable disease. Chest X-ray (B) may show adenopathy, a widened mediastinum, pulmonary or bony metastases, or signs of tracheoesophageal fistula that may be associated with esophageal carcinoma, however it is not specific for the definitive diagnosis of esophageal cancer. CT scan (C) should be obtained once the diagnosis of carcinoma has been made to evaluate for pulmonary or hepatic metastases, lymphadenopathy, and local tumor extension. Barium esophagram (A) is obtained as the first study to evaluate the dysphagia. The appearance of a polypoid, obstructive, or ulcerative lesion is suggestive of carcinoma and requires endoscopic evaluation.

An obese man presents with a 1-month history of dyspepsia. He denies dysphagia, odynophagia, vomiting, or weight loss. His exam reveals no concerning findings. You start him on antacids and order a fecal occult blood test (FOBT). He returns 1-month later unchanged. His FOBT is negative. Which of the following would be most likely used to diagnose gastroesophageal reflux disease (GERD) in this patient? Esophagogastroduodenoscopy HIDA scan High resolution manometry Proton pump inhibitor trial

Correct Answer ( D ) Explanation: Gastroesophageal reflux disease (GERD) is a common condition affecting many patients. At its core is a problem with the lower esophageal sphincter (LES), either excessive relaxation or incompetency. This leads to gastric acid erosion of the mucosa, with subsequent inflammation and stricture formation. Risk factors include obesity, hiatal hernia, overeating, delayed gastric emptying and other gastroparetic conditions, and gastric hypersecretory states. GERD can be precipitated by pregnancy, excessive caffeine, alcohol or tobacco use, fatty foods and supine positioning. It presents with heartburn, regurgitation, dysphagia and atypical chest pain. Cough, asthma and laryngitis may be other common symptoms. Diagnosis is based on a positive trial of a proton pump inhibitor (PPI). Treatment includes lifestyle modifications, PPIs, antacids and surgical (eg, Nissen fundoplication). Indications for endoscopy in patients with GERD include failure to respond to prolonged maintenance therapy, bleeding, dysphagia, or other change in symptoms. Esophagogastroduodenoscopy (EGD) (A) is used in diagnosing GERD if the patient fails to respond to an empiric trial of proton pump inhibition or if red flag features are present (dysphagia, odynophagia, weight loss, persistent vomiting, palpable mass or adenopathy or positive FOBT). Hepatobiliary iminodiacetic acid (HIDA) (B) scan is a form of cholescintography used in the diagnosis of acute cholecystitis (not GERD). High-resolution manometry (C) with 24-hour pH monitoring is used if GERD is suspected but an EGD is normal.

A 54-year-old man is unable to tolerate his saliva after feeling a piece of steak get stuck. What is the most likely cause of his dysphagia? Achalasia Barrett's esophagus Neoplasm Schatzki's ring

Correct Answer ( D ) Explanation: Impacted esophageal food boluses are a common presenting complaint of patients seen in the ED. Most food boluses pass spontaneously but 10 to 20% of them require an intervention. There are four naturally occurring levels of narrowing of the esophagus where boluses may lodge: the cricopharyngeus muscle (upper esophageal sphincter), the aortic arch, the left mainstem bronchus, and the lower esophageal sphincter. Structural abnormalities are found in up to 90% of patients with a food impaction. Schatzki's rings are fibrous strictures and the most common cause and found in up to 15% of the population. Acalasia (A) affects the ability of the esophagus to move food into the stomach since it is a failure of smooth muscle fibers to relax. It is characterized by incomplete lower esophageal sphincter (LES) relaxation, increased LES tone, and lack of peristalsis of the esophagus. Barrett's esophagus (B) is a condition resulting from chronic gastroesophageal reflux where the normal squamous epithelial cells of the esophagus are replaced with metaplastic columnar epithelial cells. Barrett's esophagus is correlated with the development of adenocarcinoma of the esophagus. Neoplasm (C) is another structural abnormality that causes dysphagia but is much less common. Adenocarcinoma is often associated with metaplasia resulting from reflux. Primary squamous cell carcinoma of the esophagus is often associated with smoking.

A 60-year-old man comes to the clinic complaining of difficulty swallowing, heartburn, chest pain, and mild weight loss. He has difficulty swallowing solids and liquids and often regurgitates undigested food. He has no past medical history and takes no medications. He denies any recent travel. Esophageal manometry reveals aperistalsis in the distal two-thirds of the esophagus and incomplete lower esophageal sphincter (LES) relaxation. Which of the following is the most likely diagnosis? Barrett's esophagus Infectious esophagitis Plummer-Vinson syndrome Primary achalasia

Correct Answer ( D ) Explanation: Primary achalasia is a disease of unknown etiology in which there is a loss of peristalsis in the distal esophagus and a failure of lower esophageal sphincter (LES) relaxation with swallowing. These patients will typically present with difficulty swallowing (dysphagia to both solids and liquids), heartburn, substernal (or retrosternal) chest pain, and regurgitation of undigested food. Esophageal manometry typically shows that pathognomonic findings of aperistalsis in the distal esophagus along with incomplete LES relaxation. Barrett's esophagus (A) is a severe complication of long-standing GERD in which intestinal metaplasia of the distal esophagus occurs. In addition to GERD symptoms, these patients also typically present with hematemesis and moderate-to-severe weight loss. Infectious esophagitis (B) is usually due to herpes simplex virus (HSV), cytomegalovirus (CMV), or Candida species. It can also be seen in immunocompromised patients. This condition does not result in aperistalsis and has no effect on the lower esophageal sphincter. Plummer-Vinson syndrome (C) consists of dysphagia, upper esophageal webs, and iron deficiency anemia.

Which of the following is recommended for the treatment of systolic heart failure according to New York Heart Association (NYHA) functional class? Anticoagulation for NYHA class I-IV heart failure Calcium channel blockers for NYHA class III-IV heart failure Hydralazine plus nitrates for white patients with NYHA class III-IV heart failure Spironolactone for NYHA class III-IV heart failure

Correct Answer ( D ) Explanation: Spironolactone for NYHA class III-IV heart failure is recommended for the treatment of systolic heart failure. Indications for specific medications for systolic heart failure are generally based on the patient's functional status as measured by the New York Heart Association (NYHA) functional class. Treatment with angiotensin converting enzyme inhibitors and beta-blockers are standard medical therapy for all patients with systolic heart failure regardless of functional stage. Spironolactone, in addition to standard medical therapy is indicated for more severe heart failure, specifically NYHA class III-IV. The addition of hydralazine plus nitrates to standard medical therapy is also indicated for NYHA class III-IV; however, only in African American patients. Anticoagulation (A) and calcium channel blockers (B) are not indicated in the treatment of heart failure, regardless of functional stage. Hydralazine plus nitrates for white patients with NYHA class III-IV heart failure (C) is incorrect because this combination is only recommended for black patients or those who cannot tolerate angiotensin converting enzyme inhibitors or angiotensin II receptor blockers.

A full term male infant is delivered by cesarean section because of dystocia due to macrosomia. Apgar scores are 8 and 10. An hour after delivery he begins to have tachypnea without hypoxemia. A chest radiograph shows diffuse parenchymal infiltrates and fluid in the pulmonary fissures. The symptoms resolve without treatment within 24 hours. What is the most likely diagnosis? Laryngomalacia Meconium aspiration syndrome Respiratory distress syndrome Transient tachypnea of the newborn

Correct Answer ( D ) Explanation: This child had transient tachypnea of the newborn, the most common cause of neonatal respiratory distress. It is a benign condition due to residual pulmonary fluid remaining in the lungs after delivery. Signs of respiratory distress such as tachypnea, nasal flaring, grunting, retractions, hypoxia, and increased oxygen requirement become evident shortly after birth. The disorder is transient, with symptoms usually resolving within 72 hours after birth. Risk factors include cesarean delivery, macrosomia, male gender, and maternal asthma and diabetes mellitus. Chest radiography is the diagnostic standard for transient tachypnea of the newborn. The characteristic findings include prominent perihilar streaking, which correlates with the engorgement of the lymphatic system with retained lung fluid, and fluid in the fissures. Small pleural effusions may be seen. Patchy infiltrates have also been described. Medical care of transient tachypnea of the newborn is supportive. As the retained lung fluid is absorbed by the infant's lymphatic system, the pulmonary status improves. Respiratory distress syndrome (C) is an acute lung disease caused by surfactant deficiency, which is primarily seen in premature infants younger than 32 weeks' gestation. Chest radiographs reveal bilateral, diffuse, or ground-glass appearances. Laryngomalacia (A) is a congenital abnormality of the laryngeal cartilage. Laryngomalacia is the most common cause of congenital stridor. Sounds typically start at age 4-6 weeks of age and in more than 90% of cases, the only treatment necessary for laryngomalacia is time. In severe cases in which the laryngomalacia interferes with ventilation enough to impair normal eating, growth, and development, a surgical approach is possible. The diagnosis of meconium aspiration syndrome (B) requires the presence of meconium stained amniotic fluid or neonatal respiratory distress, and characteristic radiographic abnormalities such as air trapping and atelectasis.

A 45-year-old man presents to the emergency department with cellulitis of his left leg. He is being treated with vancomycin. Ten minutes after the infusion is initiated, he complains of intense pruritus and lightheadedness. Erythema is noted on his face and neck. Which of the following is the next best step in the management of this patient? Administer intramuscular epinephrine and decrease the rate of the infusion Administer intramuscular epinephrine and suspend the infusion Administer intravenous diphenhydramine and decrease the rate of the infusion Administer intravenous diphenhydramine and suspend the infusion

Correct Answer ( D ) Explanation: This patient is experiencing red man syndrome, an anaphylactoid reaction to vancomycin. Clinically, patients may experience pruritus and an erythematous rash that affects the face, neck, and upper torso. Other, less common, symptoms include fever, dizziness, burning sensation, and hypotension. An anaphylactoid reaction is characterized by histamine release due to degranulation of mast cells, independent of IgE (as opposed to anaphylaxis, which is mediated by IgE). It is best treated with an antihistamine and suspension of the offending agent (vancomycin). If the symptoms resolve and there is no evidence of true anaphylaxis, the vancomycin can be restarted at a slower rate of infusion. Red man syndrome is not caused by impurities in vancomycin preparations, as it was previously thought (earning it the nickname "Mississippi mud"). Anaphylactic reactions, in contrast, are mediated by IgE and require treatment with epinephrine (A and B), and discontinuation of the offending agent. While anaphylaxis to vancomycin is possible, it is rare and red man syndrome is a much more common reaction. Both anaphylactoid and anaphylactic reactions require the offending agent be suspended, rather than just decreasing the rate of infusion (C). For anaphylactoid reactions, the infusion can resume at a slower rate after the symptoms have resolved.

Man, 28, laceration to left forearm. One centimeter. In good health, no other complaints. Pressure 155 over 94 in triage. Following repair, pressure is unchanged. Management? Administer labetalol or nifedipine with observation until systolic is <140. Evaluate for end organ damage with chest X-ray, ECG, electrolytes, BUN, creatinine and urinalysis. Follow-up with private physician within two months for recheck. No further management.

Follow-up with private physician within two months for recheck. All patients seen for emergency care, regardless of the complaint should have their blood pressure recorded. A vast majority of the hypertension encountered in the ED is either transient or mild. The most common cause of transient hypertension is pain and anxiety. In these patients, end-organ ischemia does not occur, and attention is focused on treatment of the primary process. If the pressure is elevated, it should be repeated prior to discharge. The patient is advised of this potential problem and appropriate follow up arranged. In adults, systolic blood pressure less than 120 mm Hg and diastolic pressure of less than 80 mm Hg are considered normal. If the diastolic pressure is between 80 to 104 mm Hg, the patient should have follow-up within two months. For diastolic blood pressure greater than or equal to 115 mm Hg, the patient should be evaluated immediately. The patient does not require diagnostic workup (B) or therapeutic intervention (A). The patient should be informed of his or her blood pressure and arrangements should be made for outpatient follow up (D).

What are the components of the San Francisco syncope rule?

History of CHF, hematocrit < 30%, abnormal ECG, dyspnea, systolic BP <90.

Which lab should be ordered first for suspected acromegaly? Growth hormone. Hemoglobin A1C. Insulin-like growth factor 1. Thyroid stimulating hormone.

Insulin-like growth factor 1. Acromegaly is a rare disease that occurs as a result of persistent hypersecretion of growth hormone (GH). An excess amount of GH causes hepatic secretion of insulin-like growth factor 1 (IGF-1). Excessive levels of both GH and IGF-1 cause the clinical manifestations of acromegaly including enlargement of the jaw, hands and feet, coarse facial features, macroglossia, and paresthesias of the hands. The age of onset determines height. Childhood onset prior to the fusion of the long bone epiphyses results in an increase in linear growth and pituitary gigantism. Patients with adult onset of acromegaly do not become taller. Suspicion of acromegaly merits workup with biochemical testing to confirm the diagnosis and radiologic testing to determine the cause of the abnormal levels of GH. The initial lab to determine the presence of acromegaly is IGF-1. While both IGF-1 and GH levels are elevated in patients with acromegaly, IGF-1 levels do not fluctuate throughout the day like GH levels do which makes it a more reliable initial lab. Testing of GH after administration of oral glucose should be done after elevated IGF-1 levels are determined. Pituitary MRI is the initial radiologic test ordered after elevated GH and IGF-1 levels are determined. Growth hormone (A) testing is done after determination of elevated IGF-1 levels in a patient with suspected acromegaly. Hemoglobin A1C (B) is a screening test used to determine the presence of diabetes mellitus and is not used in the workup of acromegaly. Thyroid function is generally normal in patients with acromegaly, and thyroid stimulating hormone (D) testing is done if there is suspicion of thyroid dysfunction.

Man, 80, sudden-onset left-sided abdominal pain and bloody diarrhea. CT shows thickening of bowel wall and free fluid. Diagnosis? Diverticulitis. Infectious colitis. Ischemic colitis. Small bowel obstruction.

Ischemic colitis. Ischemic colitis is the sudden reduction of blood flow to the colon usually caused by an arterial occlusion, venous thrombosis, or hypoperfusion. Ischemic colitis is the most common form of ischemic bowel disease and most often affects the elderly. Blood to the colon is supplied by the mesenteric vasculature and when this is compromised, a reduction in blood flow occurs causing mesenteric and colonic ischemia. Patients with ischemic colitis present with sudden onset of mild to severe cramping, often on the left side of the abdomen, along with rectal bleeding or bloody diarrhea within 24 hours of symptom onset. An abdominal CT may demonstrate findings of thickening of bowel wall or free peritoneal fluid and colonoscopy is diagnostic. Treatment depends on the severity and complexity of the disease, ranging from supportive care to antithrombotic therapy to surgical exploration. Complications include necrosis of the bowel and gangrene, which are medical emergencies. Diverticulitis (A) is inflammation of a sac-like protrusion of the colonic wall. Patients complain of sudden onset of pain in the left lower quadrant of the abdomen, but bloody diarrhea is uncommon with this condition. A CT of the abdomen demonstrates bowel wall thickening and the presence of diverticula, making the diagnosis of diverticulitis. Free peritoneal fluid on a CT scan is not associated with this disease. Infectious colitis (B) is the infectious process within the colon caused by bacteria, viruses, and protozoans such as salmonella, norovirus, and cryptosporidium. Patients with infectious colitis present with severe abdominal pain that may begin as quickly as six hours after ingestion of a toxin, possible fever, and differing stool characteristics from watery diarrhea to bloody stools. Obtaining a stool sample is a priority, and imaging and endoscopy are nondiagnostic. Treatment includes IV hydration and alteration of diet. A small bowel obstruction (D) occurs when there is a disruption of normal intestinal flow. Patients present with a complaint of acute abdominal cramping, distention, nausea, and vomiting. Symptoms may wax and wane, but bloody stools are an uncommon finding. An abdominal CT demonstrates bowel obstruction with dilated proximal bowel and air-fluid levels. Treatment involves bowel rest and decompression.

You are examining a newborn male. Scrotal palpation reveals the absence of a testicle. Which of the following is the most appropriate intervention? Oophorectomy. Orchipexy. Preputioplasty. Salpingectomy.

Orchipexy. Normal testicle development begins at 7 weeks gestation in the abdomen. At about week 28, they descend through the internal inguinal ring into the scrotum. An undescended testicle will remain in the inguinal canal, abdomen or retroperitoneal space. This is called cryptorchidism, which affects 3% of term and 30% of preterm infants. Nearly 67% of all undescended testes descend by the age of 4 months. About 10% of cases are bilateral. An empty scrotum and an inguinal hernia are the most common presenting signs. The diagnosis is mainly clinical, but may also require laparoscopy or ultrasonography. The necessary treatment is orchipexy, the surgical repositioning of an undescended testicle into the scrotum with subsequent hernia repair. Repairing an undescended testicle improves fertility, but does not lower the risk malignancy. Oophorectomy (A) is the surgical removal of an ovary. Preputioplasty (C) is a plastic surgical operation used to reduce a non-retractile, constricting foreskin (phimosis). Salpingectomy (D) is the surgical removal of one or both fallopian tubes.

Boy, 16. Syncope at start of track practice. ECG shows QT of 520 milliseconds. Associated with which rhythm abnormality? Paroxysmal supraventricular tachycardia. Polymorphic ventricular tachycardia. Sinus arrest. Third degree atrioventricular block.

Polymorphic ventricular tachycardia AKA Torsdes. Patients with repeated ECGs showing a QTc interval > 480 ms with a syncopal episode, or > 500 ms in the absence of symptoms, are diagnosed with long QT syndrome if no secondary cause such as medication use is present. Prolong QT interval is associated with polymorphic ventricular tachycardia, including torsades de pointes, and sudden cardiac death. Most cardiac events are precipitated by vigorous exercise or emotional stress, but they also can occur during sleep. Long QT syndrome is usually diagnosed after a person has a cardiac event such as syncope or cardiac arrest. In some situations, this condition is diagnosed after a family member suddenly dies. It may be treated with beta-blockers and implanted cardioverter defibrillators. Patients with long QT syndrome should avoid participation in competitive sports, strenuous exercise, and stress-related emotions. Epinephrine adrenaline for local anesthesia and asthma medication should be avoided in patients with long QT syndrome. Other medications that should be avoided include certain antibiotics, antifungals, antihistamines, antiarrythmics and psychotropic medications which prolong the QT interval. Third-degree atrioventricular block (D) results from various pathologic states causing infiltration, fibrosis, or loss of connection in portions of the healthy conduction system. Third-degree atrioventricular block can be either congenital or acquired. Paroxysmal supraventricular tachycardia (A) is a narrow-complex tachycardia that has a regular, rapid rhythm and is triggered by a reentry mechanism. This may be induced by premature atrial or ventricular ectopic beats. Other triggers include hyperthyroidism and stimulants, including caffeine, drugs, and alcohol. Paroxysmal supraventricular tachycardia is observed not only in healthy individuals; it is also common in patients with previous myocardial infarction, mitral valve prolapse, rheumatic heart disease, pericarditis, pneumonia, chronic lung disease, and current alcohol intoxication. Sinoatrial arrest (C) is when the sinoatrial node of the heart transiently ceases to generate the electrical impulses that normally stimulate the myocardial tissues to contract. It is defined as lasting from 2.0 seconds to several minutes. None of the above are associated with long QT syndrome.

Man, 42, with eye redness. Food poisoning last night, vomiting and wretching throughout the night. Today, noticed bleeding in eye which is focal. Plan? Anterior chamber evaluation. Coagulation studies. Complete blood count. Reassurance.

Reassurance This patient has a subconjunctival hemorrhage. These hemorrhages occur when a blood vessel ruptures under the surface of the conjunctiva. On physical examination, the hemorrhage is flat and bright red and does not cross the limbus. These hemorrhages may occur spontaneously or as a result of Valsalva maneuvers or trauma. In this case, the patient reports multiple episodes of vomiting over the night preceding recognition of the hemorrhage. The hemorrhages do not cause any long-term sequelae and treatment is aimed at reassurance. A subconjunctival hemorrhage is typically painless without any associated visual impairment or photophobia. If these conditions are present, other diagnoses should be pursued. An anterior chamber evaluation (A) is part of the complete physical examination of the eye using the slit lamp. The anterior chamber may develop inflammatory changes and debris in keratitis. Additionally, subtle hyphemas (collection of blood in the anterior chamber) may be difficult to see without inspection of the anterior chamber. Coagulation studies (B) are indicated in patients on anticoagulation therapy. However, in the routine evaluation of a subconjunctival hemorrhage, these tests are not necessary. A complete blood count (C) to measure platelets is indicated if there is concerned for thrombocytopenia in the patient. This may include patients with other areas of active bleeding, patients on chemotherapy or with marrow suppressing diseases.

Gal, 32. Pap smear performed, cytology negative. HPV-positive. Next step? Counsel patient on safe sex practices. Order colposcopy. Repeat Pap smear and HPV testing in 5 years. Repeat Pap smear and HPV testing in 1 year.

Repeat Pap smear and HPV testing in 1 year. Cervical cancer screening is done through the use of the Papanicolaou (Pap) test for cytology and testing for high-risk strains of the human papillomavirus (HPV). Initial screening begins at age 21, regardless of sexual activity. Type and frequency of testing then depend on the patient's age. Women under 30 years should be screened with the Pap smear alone every three years. Women 30 and older should be screened with Pap smear and HPV co-testing. If the Pap smear test is negative and HPV test is positive, providers may either repeat Pap smear and HPV co-testing in one year or order HPV DNA typing to detect HPV subtypes 16 or 18. These subtypes are associated with the highest risk of cervical cancer and are important to identify early, allowing for prompt intervention. All patients should be counseled on safe sex practices (A) regardless of lab results. Patients with significantly abnormal cytology require further evaluation with repeat cytology and HPV testing or colposcopy (B). Negative cytology would not require immediate referral for colposcopy. For women aged 30 years and older, the recommendation for a Pap smear with negative cytology and negative HPV is repeat co-testing in five years (C).

What is the treatment of choice for neonatal respiratory distress syndrome?

Surfactant. Transient tachypnea of the newborn shows parenchymal infiltrates and fluid in the pulmonary fissures from residual pulmonary fluid. Resolves within 72 hours.

True or false: carpal tunnel is a common complication of hypothyroidism?

True.

A patient presents with hematemesis. What test is most likely to determine the etiology of the bleeding? CT scan of the abdomen and pelvis. Nasogastric tube lavage. Right upper quadrant ultrasound. Upper endoscopy.

Upper endoscopy. Upper endoscopy is the modality that is most likely to identify the culprit lesion in a patient with upper gastrointestinal bleeding (UGIB). UGIB is a common presentation caused by a variety of pathologies including gastritis, esophageal varices, peptic ulcer disease, Mallory-Weiss tears, arteriovenous malformations and Boerhaave's syndrome. Of these causes, peptic ulcer disease is the most common. Regardless of the etiology, endoscopy represents the best modality for diagnosis. It allows direct visualization of the esophagus, stomach and first two sections of the duodenum. Additionally, it allows for interventions to be performed if active bleeding or stigmata of recent bleeding are found. CT scan of the abdomen and pelvis (A) is limited in its ability to give a diagnosis. Nasogastric tube lavage (B) may show the presence of blood in the upper GI tract but cannot differentiate between causes. Right upper quadrant ultrasound (C) may give information about the patient including the presence of cirrhosis but cannot give a specific diagnosis as the cause.


Set pelajaran terkait

Leadership in Leisure Services_CH5 Communication Skills for Leaders

View Set

Delegated, Reserved, Concurrent and Denied Powers

View Set

CIT 434 Chapters 6-14 Study Guide

View Set

Music 101 Intro to Listening Final

View Set